Merge remote branch 'public/master'
[course.git] / latex / problems / Serway_and_Jewett_4 / problem19.13.tex
1 \begin{problem*}{19.13}
2 Three point charges are arranged as shown in Figure P19.13.\\
3 %\begin{center}
4 % \begin{tabular}{|l|r|r|r|}
5 %   Name & Charge (nC) & x (m) & y (m) \\
6 %   \hline
7 %   $q_1$ & $5.00$ & $0$ & $0$ \\
8 %   $q_2$ & $6.00$ & $0.300$ & $0$ \\
9 %   $q_3$ & $-3.00$ & $0$ & $-0.100$ \\
10 %   \hline
11 % \end{tabular}
12 %\end{center}
13 \Part{a} Find the vector electric field \vect{E} that $q_2$ and $q_3$
14 together create at the origin.
15 \Part{b} Find the vector force \vect{F} on $q_1$.
16 \end{problem*} % problem 19.13
17
18 \empaddtoprelude{
19   numeric a;
20   pair A, B, C;
21   a := 3cm;
22   A := origin;   % q1
23   B := (a,0);    % q2
24   C := (0,-a/3); % q3
25   def drawB = 
26     label.top(btex 0.300\mbox{ m} etex, draw_length(B, A, 8pt));
27     label.lft(btex 0.100\mbox{ m} etex, draw_length(A, C, 8pt));
28     labeloffset := 6pt;
29     draw_pcharge(A, 4pt);
30     label.rt(btex $q_1 = 5\mbox{ nC}$ etex, A);
31     draw_pcharge(B, 4.2pt);
32     label.rt(btex $q_2 = 6\mbox{ nC}$ etex, B);
33     draw_ncharge(C, 3pt);
34     label.rt(btex $q_3 = -3\mbox{ nC}$ etex, C);
35   enddef;
36 }
37
38 \begin{nosolution}
39 \begin{center}
40 \begin{empfile}[2p]
41 \begin{emp}(0cm,0cm)
42   drawB;
43 \end{emp}
44 \end{empfile}
45 \end{center}
46 \end{nosolution}
47
48 \begin{solution}
49 \begin{center}
50 \begin{empfile}[2]
51 \begin{emp}(0cm,0cm)
52   label.lft(btex $E_{21}$ etex, draw_Efield(B, A, a/8));
53   label.rt(btex $E_{31}$ etex, draw_Efield(C, A, -a/5));
54   draw_ijhats(-(a, a/3), 0, a/6);  
55   drawB;
56 \end{emp}
57 \end{empfile}
58 \end{center}
59 \Part{a}
60 \begin{equation}
61 \begin{equation}
62  \vect{E} = k_e \sum_i \frac{q_i}{r_i^2}\rhat_i
63           = k_e \left[\frac{q_2}{x_2^2}(-\ihat) + \frac{q_3}{y_3^2}\jhat\right]
64           = 8.99\E{9}\U{N$\cdot$m$^2$/C$^2$} \left(\frac{-6.00\ihat}{0.300^2} - \frac{3.00\jhat}{0.100^2}\right)\E{-9}{C/m^2}
65           = \ans{\left( -0.599\ihat - 2.70\jhat \right)\U{kN/C}}
66 \end{equation}
67
68 \Part{b}
69 \begin{equation}
70  \vect{F} = q_1 \vect{E}
71           = \ans{\left( -3.00\ihat -13.5\jhat\right)\U{$\mu$N}}
72 \end{equation}
73 \end{solution}